Đến nội dung

nth1235

nth1235

Đăng ký: 24-02-2012
Offline Đăng nhập: 25-11-2015 - 19:42
*----

#426660 Đề thi vào lớp 10 THPT chuyên Lương Thế Vinh, Đồng Nai 2013-2014 (toán chuyên)

Gửi bởi nth1235 trong 13-06-2013 - 09:19

Đề năm nay dễ hơn rất nhiều so với đề năm ngoái,




#357570 Chứng minh rằng luôn có 1 trong 2 tập đó chứa 3 số sao cho tổng của 2 trong...

Gửi bởi nth1235 trong 29-09-2012 - 20:09

1. Chứng minh rằng : với mọi chia tập $ X= {1 ; 2 ; 3 ; 4 ; 5 ; 6 ; 7 ; 8 ; 9} $ thành hai tập con, luôn có 1 trong 2 tập đó chứa 3 số sao cho tổng của 2 trong 3 số đó bằng 2 lần số còn lại.
2. Tồn tại hay không 2 tập vô hạn A, B chứa các số nguyên không âm sao cho với số không âm $c$ bất kì nào đó, đều tìm được $a \in A , b \in B$ sao cho $c = a + b$.
  • LNH yêu thích


#353241 Tính: $\sqrt{2\sqrt{2\sqrt{2\sqrt...

Gửi bởi nth1235 trong 09-09-2012 - 19:57

Cái này bạn sẽ được học rõ hơn khi học sang năm lớp 11. Chính xác là phần lim. Còn bây giờ bạn có thể hiểu nôm na rằng vì số căn của nó quá nhiều nên khi bớt một dấu căn thì giá trị của nó không thay đổi.


#351385 [MO2013] Trận 1 - Phương trình, hệ phương trình, bất phương trình

Gửi bởi nth1235 trong 01-09-2012 - 16:05

Giải hệ phương trình trên tập hợp số thực :
$\begin{cases}
& \text \sqrt[8]{2.\sqrt[5]{7} - \sqrt[10]{y}} + (17 - \sqrt{37}).z^2 = 544 - 32.\sqrt{37} \\
& \text x.(9.\sqrt{1 + x^2} + 13.\sqrt{1 - x^2}) + 4\sqrt{y} = 912 \\
& \text \sqrt{(10.\sqrt{5} + 20).x.(1 - x)} + z.\sqrt[6]{8} = 10
\end{cases}$

BL :
ĐK : $0 \leq x \leq 1 ; 0 \leq y \leq 50176 (1)$
Với $0 \leq x \leq 1$, ta có :
$ x.(9.\sqrt{1 + x^2} + 13.\sqrt{1 - x^2})
= \frac{3}{2} . 3x . 2\sqrt{1 + x^2} + \frac{13}{2} . x . 2\sqrt{1 - x^2}
\leq \frac{3}{4}.[9x^2 + 4(1 + x^2)] + \frac{13}{4}.[x^2 + 4(1 - x^2)] = 16 (2)$ (Áp dụng BĐT AM - GM)
Dấu "=" xảy ra $\Leftrightarrow x = \frac{2}{\sqrt{5}} $ (thỏa $(1)$)
Mặt khác, từ $(1)$ suy ra $4.\sqrt{y} \leq 896 (3)$.
Từ $(2) , (3)$ suy ra $x.(9.\sqrt{1 + x^2} + 13.\sqrt{1 - x^2}) + 4.\sqrt{y} \leq 912$.
Mà theo đề bài, dấu bằng xảy ra nên $x = \frac{2}{\sqrt{5}} ; y = 50176$
Từ đó, thay $x = \frac{2}{\sqrt{5}} ; y = 50176$ vào hệ, suy ra $z = \sqrt{32}$
Vậy hệ có nghiệm $(x , y , z)$ duy nhất là $(\frac{2}{\sqrt{5}} ; 50176 ; \sqrt{32})$

Nhận xét : Thoạt nhìn hệ trên có vẻ khá phức tạp nhưng nếu chứng minh được bất đẳng thức $x.(9.\sqrt{1 + x^2} + 13.\sqrt{1 - x^2}) \leq 16 $ với $ 0 \leq x \leq 1$ thì bài toán trở nên đơn giản hơn.

PS : Ban tổ chức giúp em sửa Latex 1 số chỗ dấu căn hiển thị chưa rõ với. Máy nhà em ko hiểu sao sửa mãi mà không được.

OK


#350110 [MO2013] Trận 1 - Phương trình, hệ phương trình, bất phương trình

Gửi bởi nth1235 trong 27-08-2012 - 09:10

Đề vừa ra đã có bạn đưa lên mathlinks mong BQT xem xét

Là ai bạn có thể nói rõ ko ???


#350109 [MO2013] Trận 1 - Phương trình, hệ phương trình, bất phương trình

Gửi bởi nth1235 trong 27-08-2012 - 09:03

Thống kê bao nhiêu bạn làm dc bài : (Theo thứ tự thời gian)
1) bibitsubomi 9fxshiftsolve
2) WhjteShadow
3) henry0905
4) davildark
5) hoangtrunghieu22101997
6) nthoangcute
7) triethuynhmath
8) minhtuyb
9) milinh7a
10) doxuantung97
11) chinhanh9
12) L Lawliet
13) namheo1996
14) sherlock holmes 1997
15) Trần Đức Anh @@
16) tran thanh binh dv class
17) 899225
18) luuxuan9x
19) Joker999
20) ninhxa.
Ngoài ra còn có 1 mở rộng của WhjteShadow.


#348475 Topic các bài toán số học dành cho các bạn chuẩn bị thi tuyển sinh 10 năm 201...

Gửi bởi nth1235 trong 20-08-2012 - 09:30

Bài 21: http://diendantoanho...m-2006m12006n1/
Bài 20: đã có trên VMF

Chán thế, sao bài nào cũng có vậy, bài khác.
Bài 22 : Cho $m, n$ là các số nguyên dương thỏa mãn :
$ lcm(m , n) + gcd(m , n) = m + n$
Chứng minh rằng trong hai số $m , n$ có một số chia hết cho số còn lại.
Bài 23 : Tìm tất cả các số tự nhiên $n$ sao cho : $A = {n}^{2005} + {n}^{2006} + n^2 + n + 2$ là số nguyên tố.


#348368 Topic các bài toán số học dành cho các bạn chuẩn bị thi tuyển sinh 10 năm 201...

Gửi bởi nth1235 trong 19-08-2012 - 20:32

Bài 20 : Chứng minh rằng số được thành lập bởi $3^n$ chữ số giống nhau thì chia hết cho $3^n$, trong đó $n$ là một số nguyên dương cho trước.
Bài 21 : Cho $m, n$ là hai số nguyên dương phân biệt có $(m , n) = d$ $(d$ là số nguyên dương$)$.
Tính $(2006^m + 1 , 2006^n + 1)$


#347951 Topic các bài toán số học dành cho các bạn chuẩn bị thi tuyển sinh 10 năm 201...

Gửi bởi nth1235 trong 18-08-2012 - 18:26

Cho mình hỏi câu này, tập hợp $P$ là tập hợp gì mà nghe lạ quá @@
@nguyenta98:

Spoiler

Tập hợp $P$ là tập hợp số nguyên tố.
Ps : Sao toàn người quen giải bài không vậy ??? Đề nghị "Cao thủ số học" Tạ Hà Nguyên ngừng việc post lời giải lên nữa, phải để cho mấy bạn khác làm với chứ. :D


#347782 Topic các bài toán số học dành cho các bạn chuẩn bị thi tuyển sinh 10 năm 201...

Gửi bởi nth1235 trong 18-08-2012 - 07:52

Post mấy bài góp vui :
$11)$ Cho $a, b, c, d$ là các số nguyên dương thỏa mãn $(a , b) = (c , d) = 1.$
Chứng minh nếu $\frac{a}{b} + \frac{c}{d}$ là số nguyên thì $b = d$.
$12)$ Tìm số có hai chữ số $\bar{ab}$ sao cho $ p = \frac{a.b}{\left| a - b \right|}$ là số nguyên tố.


#347779 Topic các bài toán số học dành cho các bạn chuẩn bị thi tuyển sinh 10 năm 201...

Gửi bởi nth1235 trong 18-08-2012 - 07:23

Từ đầu bài suy ra: $A=(2005^n-1897^n)-(168^n-60^n)$
Ta có:
$2005^n-1897^n \vdots 2005-1897=108 \vdots 12$
$168^n-60^n \vdots 168-60=108 \vdots 12$
$\Rightarrow A \vdots 12$ (1)
Từ đầu bài ta cũng có: $A=(2005^n-168^n)-(1879^n-60^n)$
Ta có:
$2005^n-168^n \vdots 2005-168=1837 \vdots 167$
$1897^n-60^n \vdots 1897-60=1837 \vdots 167$
$\Rightarrow A \vdots 167$ (2)
Mà $2004=12.167$ (3)
Từ (1), (2) và (3) $\Rightarrow A \vdots 2004$
----

Spoiler

Bài này bạn nên chú ý chỗ (3). Chỗ đó chỉ đúng khi (12,167) = 1. Bạn nên lý luận thêm.
Một vd để bạn thấy nó sẽ ra sao nếu ko có điều kiện đó :
4 chia hết cho 2 và 4 chia hết cho 4 nhưng không chia hết cho 2.4 = 8.


#347399 Chứng minh $(p - 2)!$ chia hết cho $(p - 1)$

Gửi bởi nth1235 trong 17-08-2012 - 07:50

Một cách làm khác mà tối qua mình nghĩ ra.
Giả sử điều ngược lại, $p$ là hợp số.
Xét hai trường hợp :
TH1 : $p = a^2$ ($a$ là số nguyên dương lớn hơn 1)
Khi đó, ta có : $ 1 < a < p $
Vì $a$ là số nguyên dương lớn hơn 1 nên $a^2 - a - 1 > 0$.
Hay $p - 1 > a$. Do đó $ (p - 1)! $ chia hết cho $a$. $(1)$
Mặt khác, theo giả thiết thì $ (p - 1)! + 1 $ chia hết cho $ p = a^2 $.
Suy ra $ (p - 1)! + 1 $ chia hết cho $a$ $(2)$
Từ $(1)$, $(2)$ suy ra 1 chia hết cho $a$, mâu thuẫn.
TH2 : $p = x.y$ ($x, y$ là các số nguyên dương khác nhau lớn hơn 1).
Không mất tính tổng quát, giả sử $ 1 < x < y < p $
Khi đó, hiển nhiên $ p - 1 \geq y > x $. Suy ra $(p - 1)!$ chia hết cho $x$. $(1)$
Mặt khác, theo giả thiết thì $ (p - 1)! + 1 $ chia hết cho $ p = x.y $.
Suy ra $ (p - 1)! + 1 $ chia hết cho $x$ $(2)$
Từ $(1)$, $(2)$ suy ra 1 chia hết cho $x$, mâu thuẫn.
Vậy ta có điều phải chứng minh.


#346529 Chứng minh rằng: $\frac{a^{2}}{b}+\frac{b^{2}}{c}+\frac{c...

Gửi bởi nth1235 trong 13-08-2012 - 18:56

Nếu vậy cách của bạn cũng phải cần a,b,c >0 chứ nhỉ.Nếu không thì không làm được.Vậy sao bạn nói "ẩu"

Bạn chưa hiểu ý mình nói rồi. Mình nói như vậy là ý mình muốn thêm cái đk $a, b, c > 0$ vào giả thiết và sau đó làm như hai bạn kia hoặc như mình thôi chứ không có ý gì hết. "Ẩu" ở đây không có nghĩa là làm sai mà có nghĩa là vì bài toán này quen thuộc nên không chú ý đến điều kiện phải có của biến. Mình không có ý chê ai dở cả.


#344824 Chứng minh rằng tồn tại vô số số nguyên tố dạng $4k+3$ ($k...

Gửi bởi nth1235 trong 08-08-2012 - 20:09

Một cách làm khác :
Giả sử có hữu hạn số nguyên tố có dạng $4k + 3$ là $p_1;p_2;p_3;...;p_n$.
Xét số $q = 4p_1.p_2....p_n + 3$
Hiển nhiên $q$ không chia hết cho 4 và không chia hết cho 3.
Do đó, $q$ có ít nhất một ước nguyên tố $p = 4m + 3$ (chỗ này cm tương tự bạn kia). Do đó, tồn tại $i$ thuộc ${1,2,3,...,n}$ sao cho $ p = p_i$.
Vậy $4.p_1.p_2...p_n$ chia hết cho $p$ mà $q$ chia hết cho $p$ nên 3 chia hết cho $p$. Vì $p$ là số nguyên tố nên $p = 3$, mâu thuẫn (Do $q$ không chia hết cho 3).


#341466 Tìm Min của $2ab + 2ac + 2bc - a\sqrt{bc} - b\sqrt...

Gửi bởi nth1235 trong 29-07-2012 - 14:44

Một bài khá kì lạ nhỉ? :P
$a\sqrt{bc}+ b\sqrt{ac}+ c\sqrt{ab}=\sqrt{ab}.\sqrt{ca}+\sqrt{ab}.\sqrt{bc}+\sqrt{ca}.\sqrt{bc}\leq ab+bc+ca$
$\Leftrightarrow -(a\sqrt{bc}+ b\sqrt{ac}+ c\sqrt{ab})\geq -(ab+bc+ca)$
Vậy ta có: $VT=ab+bc+ca\geq 3\sqrt[3]{(abc)^{2}}=1$
Dấu "=" xảy ra khi $a=b=c=1$

Min bằng 3. Bạn viết lộn rồi.